Civil Procedure Missed Q

Réussis tes devoirs et examens dès maintenant avec Quizwiz!

In a negligence action properly before a federal district court sitting in diversity, the court submitted the case to the jury. The jury's decision, which took the form of a written special verdict, was read aloud by the court clerk in open court. The verdict stated that both parties were negligent and that both parties' negligence proximately caused the plaintiff's injuries. The verdict also stated that the plaintiff had suffered damages of $1 million and was 10% at fault for his injuries. The court then asked the jury collectively if this was their verdict and they responded in unison, "Yes." The defendant requested that the jury be polled. When questioned individually, a juror tearfully stated that the verdict was not her verdict because she did not believe that the defendant had been negligent. Upon further questioning, she maintained this position. The other seven jurors affirmed the verdict. The defendant moved for a new trial. Is the court likely to grant the defendant's motion? No, because at least six jurors agreed with the verdict. No, because a juror cannot recant a special verdict once it has been read aloud by the court clerk in open court. Yes, because the court must order a new trial when polling the jury reveals that the verdict is not unanimous. Yes, because one juror did not affirm the special verdict as hers.

Answer choice D is correct. Under the federal rules, the verdict of a civil jury must be unanimous unless the parties otherwise agree. Here, one juror, when questioned individually, denied that the special verdict was her verdict. Consequently, the court may order a new trial. Answer choice A is incorrect. Although the federal rules permit a civil jury with as few as six members, the rules require that the verdict of a civil jury be unanimous unless the parties have otherwise agreed. Answer choice B is incorrect because any verdict, including a special verdict, is subject to impeachment by a juror prior to being excused by the court, even though the verdict has been announced in open court. Answer choice C is incorrect. When polling reveals that the verdict is not unanimous, the court is not required to order a new trial, but may instead order the jurors to deliberate further. Accordingly, answer choice D is the best answer.

A commercial landlord from State A brought an action in federal district court to recover six months of overdue rent from a corporation incorporated and with its principal place of business in State B. The total amount owed under the terms of the parties' lease agreement was $80,000. After being properly served, the corporation refused to respond to the landlord's complaint. Three months later, the landlord submitted an affidavit to the court clerk attesting to the fact that the corporation failed to respond to the complaint and has not defended against the landlord's claim in any way. Accordingly, the court clerk entered the corporation's default. After default was entered, the landlord requested that the court clerk enter a default judgment against the corporation. Along with his request, the landlord submitted an affidavit indicating the amount due and owing under the terms of the lease agreement, $80,000. The court clerk then entered a default judgment against the corporation for $80,000. Was the court clerk's entry of the default judgment proper? No, because the landlord must petition the court to obtain a default judgment. No, because the court clerk must seek court approval before entering a default judgment. Yes, because the evidence presented shows no genuine issue as to any material fact. Yes, because the relief sought by the landlord was a sum certain, set forth in his affidavit.

Answer choice D is correct. When a party has failed to plead or otherwise defend an action, and that failure is shown by affidavit or otherwise, the court clerk must enter the party's default. Once a default is entered against a party, the plaintiff may seek a default judgment. If the relief sought is a sum certain or an amount that can be made certain by computation, the default judgment can generally be entered by the court clerk on the plaintiff's request, accompanied by an affidavit showing the amount due. Otherwise, the plaintiff must apply to the court for a default judgment. In this case, the landlord accompanied his request for a default judgment with an affidavit indicating the amount due and owing under the terms of the lease agreement. Because the relief sought was a sum certain, $80,000, the court clerk properly entered the default judgment against the corporation. Answer choice A is incorrect. The landlord did not need to petition the court to obtain a default judgment because the relief sought by the landlord was a sum certain. Accordingly, the court clerk could enter the default judgment. Answer choice B is incorrect. The court clerk does not need to seek court approval before entering a default judgment when the relief sought is a sum certain. Answer choice C is incorrect because it paraphrases the standard for granting a summary judgment, not a default judgment.

A farmer filed an action in federal district court based on diversity jurisdiction seeking damages attributable to a malfunctioning irrigation system in which four different defendants had participated in the design, manufacture, and installation. The farmer advanced three alternative theories of liability against the defendants—breach of contract, breach of warranty, and negligence. Prior to trial, the court required the parties to submit requests for jury instructions. Each party submitted such instructions. Prior to the close of evidence, the court held a charge conference to discuss the form of the verdict and related jury instructions. Due to the number of defendants and variety of theories of liability against each, the court determined that a special verdict form would be used. Concerned that the special verdict form failed to clearly indicate that, if the jury found for the farmer, the farmer was entitled to a single recovery of his damages, the defendants attempted to submit additional jury instructions. Is the court required to permit the defendants to submit these instructions? No, because the court had provided the defendants with the opportunity to submit jury instruction requests. No, because a court may, but is not required to, permit a party to request specific jury instructions. Yes, because the conference was held before the close of evidence. Yes, because the defendants could not reasonably have anticipated the need for such instructions prior to seeing the special verdict form.

Answer choice D is correct. Although the court may require the parties to file jury instructions at the close of evidence or at any earlier reasonable time, a party can file a request for jury instructions on issues that could not have reasonably been anticipated as of the earlier time set by the court for such requests. Here, because the defendants did not know of the special verdict form until the charge conference, it is unlikely that they could reasonably have anticipated the possible jury confusion that the defendants' requested instructions sought to address. Consequently, the defendants had a right to request jury instructions regarding the farmer's right to a single recovery. Answer choice A is incorrect. Although the court may require the parties to file jury instructions at the close of evidence or at any earlier reasonable time, a party may file a request for jury instructions on issues that could not have reasonably been anticipated as of the earlier time set by the court for such requests. Answer choice B is incorrect because Federal Rule of Civil Procedure 51 specifically permits a party to file requests for jury instructions with the court. Answer choice C is incorrect because Rule 51 permits the court to fix a reasonable time earlier than the close of evidence for the submission of requests for instructions. Here, the court did set an earlier time.

A jeweler who is a citizen and resident of a foreign country brought some jewels into the United States to sell at a convention. A buyer purchased the jewels from the jeweler, whom he had not previously met, but whom he knew by reputation to be an honest businessperson. Subsequently, the buyer discovered that the jeweler had misrepresented the quality of the jewels. The buyer has filed an action in the federal court for the district in which he resides against the jeweler based on a state law claim of misrepresentation. The convention had been held in another state. The jeweler has timely filed a motion to dismiss this action based on lack of subject matter and personal jurisdiction, as well as improper venue. The court properly determined that it has subject matter and personal jurisdiction. Should the court grant the motion as it relates to improper venue? No, because a nonresident of the United States may be sued in any judicial district. No, because the action is based on diversity jurisdiction, not federal-question jurisdiction. Yes, because the action is based on a state law claim. Yes, because the cause of action arose in another state.

Answer choice A is correct. A defendant who is not a resident of the United States may be sued in any judicial district. Because the jeweler is a resident of a foreign country, the jeweler may be sued in the federal district court for the district in which the buyer resides. Answer choice B is incorrect because, although the action is based on diversity jurisdiction because it arises from a state law claim, there is no distinction for venue purposes between actions based on diversity jurisdiction and those based on federal-question jurisdiction. Answer choice C is incorrect because, although the action is based on a state law, this fact by itself has no relevance to the question of improper venue. Answer choice D is incorrect because, although the cause of action did arise in another state, a nonforum state, venue is proper where the defendant resides and, for a defendant who is not a resident of the United States, the defendant is deemed to be a resident of any judicial district.

Family members of a deceased individual properly filed an action for money damages in federal court against a funeral home for negligent infliction of emotional distress based on the alleged mishandling of the remains of the deceased. The funeral home timely and properly served an answer to the complaint. Thirty days later, including eight weekend days, the family members served a demand for a jury trial on the funeral home. The demand was served on a holiday. On the following day, a non-holiday weekday, the family members filed the jury trial demand with the court. Was the family members' demand for a jury trial timely? No, because the demand for a jury trial was not served on the funeral home within 14 days of service of the answer. No, because the demand for a jury trial was not filed with the court within 21 days of service of the answer. Yes, because the last day for making the demand was a holiday. Yes, because weekend days are not counted in determining whether the demand was timely.

Answer choice A is correct. A demand for a jury trial must be served within 14 days after service of the last pleading directed to the issue that is sought to be tried by a jury. Here, the complaint sought damages, which is an action at law for which a jury trial may be had. However, the demand was not made within 14 days of service of the funeral home's answer, which is the only responsive pleading the funeral home may make that is directed to the plaintiff's complaint. Therefore, the demand for a jury trial was not timely. Answer choice B is incorrect. While a defendant has 21 days to respond to a complaint, a demand for a jury trial must be served on the other parties within 14 days after service of the last pleading directed to the issue for which the jury trial is sought. Answer choice C is incorrect. In computing whether a party has complied with the rules for undertaking an action within the specified time period, when the last day of the period would otherwise fall on a holiday, the action may be undertaken on the next non-holiday weekday. However, in this case, because the time period is 14, not 21 days, the time period for making a jury trial demand expired prior to the family members' service of the jury trial demand. Answer choice D is incorrect. Weekend days are counted in determining whether a party has taken action, such as making a demand for a jury trial, within the permissible time period.

A state employee was terminated from her employment. She believed the termination was based on a male supervisor's bias against female employees. The employee had no work performance issues on record. She filed suit against the state employer in federal district court based on wrongful termination in violation of the Equal Protection Clause of the Fourteenth Amendment. At the time she filed the suit, there were four other pending lawsuits against her employer, filed in state court in the state where she lived and worked, for a variety of termination issues. In light of the pending state court actions, the federal court Must still adjudicate the employee's claim. May adjudicate the employee's claim. Must stay the matter pending the adjudication of the state claims. May transfer the case to the state court.

Answer choice A is correct. A federal court with subject-matter jurisdiction is required to adjudicate the controversy despite the pendency of a similar action in a state court. Here, the federal court has subject-matter jurisdiction, as the employee is suing the state employer based on a violation of the Equal Protection Clause. There are limited circumstances in which a federal court may abstain from hearing a case pending the outcome of the state court action, but none of those circumstances are present here. Answer choice B is incorrect because the federal court is required to adjudicate the claim, as none of the limited circumstances apply. Answer choice C is incorrect because the federal court must adjudicate the claim; it cannot stay the matter pending the adjudication of the state claims. Answer choice D is incorrect because the federal court must adjudicate the claim itself. Moreover, a federal court does not have the power to transfer a case to state court.

A plaintiff filed a complaint in state court against a defendant, alleging damages from negligence resulting from an automobile accident. The defendant removed the case to federal district court. The federal district court, finding that there was no subject-matter jurisdiction in federal court, issued an order remanding the case to state court. The defendant believes that the court's remand is improper and wants to appeal the order remanding the case to state court. Which of the following is most accurate regarding the remand order? Appeal of the remand order is improper. Once a case is removed, remand is improper. Review of a remand order to state court is proper only upon the issuance of a final judgment. The remand order is appealable if the application for review is made no more than 10 days after entry of the order.

Answer choice A is correct. A remand order is generally not reviewable on appeal or otherwise (except for an order remanding a civil rights case removed pursuant to § 1443 or a remand order in a class action, if the application for review is made to the court of appeals not more than 10 days after the entry of the order). Thus, appeal of the remand order would be improper. Answer choice B is incorrect. Remand of a removed case is proper under a variety of circumstances, including, as here, when the court finds that it lacks subject-matter jurisdiction over the case. Answer choice C is incorrect. Generally, a remand order is not reviewable by an immediate appeal or otherwise. Answer choice D is incorrect. The 10-day time limit is only applicable to a remand order in a class action. In general, a remand order is not appealable.

A federal district court denied certification of an action as a class action. In doing so, the court made a mistake of law. Thirteen days after the entry of the district court's order, the plaintiff filed a petition with the clerk of the applicable circuit court for permission to appeal the denial of certification. Must the appellate court hear this appeal? No, because entertaining this appeal is at the discretion of the appellate court. No, because the petition was not timely filed. Yes, because the district court denied rather than granted certification of the class. Yes, because the district court made an error of law.

Answer choice A is correct. An appellate court may, but is not required to, hear an immediate appeal of a certification decision rendered by the district court. Answer choice B is incorrect. A petition for permission to appeal a certification order must be filed within 14 days of the entry of the order. Here, the petition was filed 13 days after the entry of the district court's order. Thus, it was timely filed. Answer choice C is incorrect because a petition for permission to appeal may be made whether the district court grants or denies certification of the class. Answer choice D is incorrect. While the commission of an error of law could be grounds for overturning the district court's order, it does not compel the appellate court to hear the appeal.

An individual wanted to become a member of an online service for farmers looking to date other farmers. In order to sign up for the online service, an individual must pay a $30 monthly membership fee and be a farmer. The individual has been denied the right to use this service on the grounds that she is not a farmer. The individual has filed suit in federal district court against the online service, alleging that she is the victim of unconstitutional discrimination. At trial, the individual testifies that her primary income comes from buying and selling beef cattle. However, in her deposition, the individual stated that she makes ceramic cows for a living. Can the defendant online service admit the individual's deposition testimony at trial? Yes, to impeach the individual's testimony at trial or for other purposes. Yes, but only to impeach the individual's testimony regarding her occupation. No, because the individual is not unavailable and could be cross-examined. No, because the individual testified at trial that she buys and sells beef cattle for a living.

Answer choice A is correct. Depositions may be used by a party to impeach the testimony of the deponent as a witness or for any other purpose permitted by the Federal Rules of Evidence. Rule 32(a)(2). Thus, the online service can use the individual's deposition to impeach her testimony at trial that she buys and sells beef cattle instead of making ceramic cows for a living. Answer choice B is incorrect because in addition to impeachment, the individual's testimony can be used for any other purpose permitted under the Federal Rules of Evidence. Answer choice C is incorrect. Although a deposition can be used in specific situations when a witness is not available to testify, it can also be used for other purposes as discussed above. Answer choice D is incorrect because a deposition can be used to impeach the testimony of the deponent as a witness at trial.

The producer of an organic pet food shipped his products nationwide. His business and production offices were located in State A, the state in which he lived. After one pet owner's animal became sick due to toxins that had leaked into the producer's production supply, the owner filed a lawsuit in federal district court based on diversity jurisdiction in State B, the state in which he lived. A federal food regulation act governed the processes for how nutrition, including that applicable to animals, should be regulated. The producer did not meet those regulations, but did meet the lower standards required by State B. If the district court applied the State B statute, the producer would be absolved of any liability. Must the district court enforce the federal statute? Yes, because the federal statute regulates nutrition as it relates to pets. Yes, because the failure to apply the federal law would result in the producer's reduced liability. No, because the failure to apply the state law would result in the producer's increased liability. No, because the court has the discretion to apply the federal law.

Answer choice A is correct. If the applicable state and federal laws conflict, the district court must ask whether a valid federal statute covers the disputed issue. Here, there is a valid federal statute on point, so the district court must apply federal law rather than the State B law. Answer choices B and C are incorrect because if there had been no federal statute on point, then the court would determine whether it should apply federal common law by asking whether the application of state law would lead to different outcomes in federal and state court. Here, however, there is a federal statute on point, so the court should apply that law, rather than the conflicting State B law. Answer choice D is incorrect because the district court must apply the federal law.

A plaintiff filed a defamation action against a defendant in federal district court under diversity jurisdiction eleven-and-a-half months after the alleged defamation occurred. The plaintiff mailed a notice of the commencement of the action along with a request for waiver of the service of the summons to the defendant, who lived in another state. The defendant signed the waiver and returned it to the plaintiff. The plaintiff filed the waiver with the court 30 days after sending the waiver request to the defendant. Twenty-three days later, the defendant filed a motion to dismiss for failure to state a claim upon which relief could be granted. The defendant contended that the action was not brought within the one-year time period for defamation actions under the applicable state law. For purposes of this and other statutes of limitations, the forum state's procedural rules provide that an action does not commence until process has been served. How should the court rule on the defendant's motion to dismiss? Grant the motion, because the statute of limitations period had expired. Grant the motion, because the federal rules govern even in a diversity action. Deny the motion, because the motion to dismiss was not timely. Deny the motion, because the action was filed within the one-year statute of limitations period.

Answer choice A is correct. If the defendant agrees to waive service, then the date on which the plaintiff files the waiver form with the court will be deemed the date of service of process. In this case, because the date the plaintiff filed the waiver form was more than one year after the alleged defamation occurred and the cause of action arose, the court should grant the defendant's motion to dismiss. Accordingly, answer choice D is incorrect. Answer choice B is incorrect. Under Rule 3, a civil action is generally commenced in federal court by filing a complaint with the court clerk. However, for the purposes of a federal diversity action, state law will apply to decide when the action commenced for the purposes of the statute of limitations. Here, the state law provides that an action is commenced by service of process on a defendant, rather than by filing with the court. Thus, the state rule will control for the purposes of diversity jurisdiction, and the motion should be granted. Answer choice C is incorrect because the defendant had 60 days after the request was sent to file an answer or pre-answer motion to dismiss; here, the defendant filed a motion to dismiss 53 days (30 + 23) after the request was sent.

A driver brought an action for negligence against a motorcyclist in federal district court arising out of a car accident. The motorcyclist collided with the driver's car after he was unable to brake in the rain while driving down a hill. At the close of all evidence, the motorcyclist moved for judgment as a matter of law. The court denied the motion and submitted the case to the jury. Following a brief deliberation, the jury returned a verdict in favor of the driver, assessing $100,000 in damages. The court then entered judgment against the motorcyclist. Six months later, the manufacturer of the motorcycle issued a product recall for the brakes. The manufacturer had discovered that the brakes did not operate in the rain if the motorcycle was being driven downhill. Neither the manufacturer nor the motorcyclist was aware of this defect during the motorcyclist's trial, despite examination of the motorcycle by experts. At this point, what is the appropriate motion for the motorcyclist's lawyer to file? Relief from judgment. Correction of a judgment. New trial. Judgment as a matter of law (renewed).

Answer choice A is correct. Rule 60(b) allows a court to relieve a party from a final judgment or order for a motion filed within a reasonable time, and no later than one year following the entry of the judgment or order, for newly discovered evidence that could not have been earlier discovered with reasonable diligence. In this case, the manufacturer issued a product recall for the brakes on the motorcyclist's motorcycle because it discovered that the brakes did not operate in the rain if the motorcycle was being driven downhill. The defect in the brakes was not discovered during the motorcyclist's trial even though experts had examined the motorcycle. The recall was made six months after judgment was entered against the motorcyclist. Accordingly, the motorcyclist's lawyer should file a motion for relief from judgment. Answer choice B is incorrect. Rule 60(a) allows a court to correct a clerical or other mistake resulting from oversight or omission whenever one is found in a judgment, order, or other part of the record. This motion is not applicable in this case. Answer choice C is incorrect. Although a motion for a new trial may be made as the result of newly discovered evidence, it must be filed no later than 28 days after the entry of judgment. In this case, six months have passed since the entry of judgment against the motorcyclist. Answer choice D is incorrect. A renewed motion for judgment as a matter of law must be filed no later than 28 days after the entry of judgment. Accordingly, this motion would also be time barred.

During the discovery phase of a trial for fraudulent misrepresentation by a corporate seller of real property, the court ordered the corporate defendant to produce emails that had been exchanged between its employees prior to the sale of the property. The defendant informed the court that the emails had been irretrievably lost. After holding a hearing, the court concluded that the emails had been lost because the defendant failed to take reasonable steps to preserve them, that the information could not be restored or replaced, and that the plaintiff had suffered prejudice as a consequence. Without a specific request by the plaintiff, the court ordered that the defendant could not present any evidence as to its employees' discussions prior to the sale of the property. The plaintiff was successful at trial. On appeal, the defendant challenged the court's order. If the appellate court rules in the defendant's favor, which of the following is the most likely reason? The trial court abused its discretion in entering the order against the defendant. The plaintiff did not request that the court impose this sanction. The defendant's conduct could not subject the defendant to this sanction. A court may not impose a sanction for a party's failure to provide electronically stored information that has been lost.

Answer choice A is correct. The court's decision to order a sanction against the defendant is subject to review under the abuse of discretion standard. Here, in ruling in favor of the defendant, the appellate court must have found that the trial court abused its discretion by prohibiting the defendant from introducing any evidence relating to its employees' discussions prior to the sale of the property. Answer choice B is incorrect. Although a party generally may not seek sanctions against another party for a discovery failure unless the party seeking the sanctions first seeks a court order, once the court has issued such an order, the court generally has discretion in the sanctions it chooses to impose if the party subject to the order fails to comply with it. The party who sought the court order need not request a specific sanction before the court may impose it. Answer choice C is incorrect. If electronically stored information that should have been preserved in the anticipation or conduct of litigation is lost because a party failed to take reasonable steps to preserve it, it cannot be restored or replaced through additional discovery, and the other party has suffered prejudice, the court may order measures no greater than necessary to cure the prejudice. There are many different measures (i.e., sanctions) that the court may order, including prohibiting a party from introducing certain evidence. Answer choice D is incorrect because Rule 37 permits sanctions to be imposed for the loss of electronically stored information if the party failed to take reasonable steps to preserve it and it cannot be restored or replaced, provided that another party is prejudiced as a result.

A plaintiff filed a complaint in federal court alleging that a cell phone application sold by the defendant infringed upon a patent held by the plaintiff. The complaint was signed by an associate at the large law firm that represented the plaintiff. The defendant filed a motion to dismiss, arguing that the cell phone application at issue did not actually perform as described in the complaint, and thus did not affect the plaintiff's patent. The court held a hearing on the motion to dismiss. At the hearing, the defendant presented evidence that the application performed only functions not covered by the plaintiff's patent. The defendant also showed that the plaintiff's attorney had never actually used the application, but had drafted the complaint based solely on his client's description of the application. The court granted the motion to dismiss. The court also issued an order requiring the plaintiff's attorney and his law firm to pay the defendant's attorney's fees, finding that the plaintiff's attorney had not conducted a reasonable inquiry into the factual contentions in the complaint. What of the following is the best argument that the court erred in its order requiring payment of attorney's fees? A court may not impose sanctions against a law firm unless the pleading at issue is signed by a partner. A court may not order a party to pay the opposing party's attorney's fees except upon motion. An attorney may rely upon factual contentions put forth by the client in a pleading. Attorney's fees are not a permissible type of sanction under Rule 11.

Answer choice B is correct. A court may impose sanctions, including payment of attorney's fees, on attorneys, law firms, and parties for violations of Rule 11, but may typically do so only when a party makes a motion for such fees. In this case, the defendant did not make a motion requesting attorney's fees, and thus the court erred in imposing such sanctions. Answer choice A is incorrect because, absent exceptional circumstances, a law firm must be held jointly responsible for a violation committed by its partner, associate, or employee. Answer choice C is incorrect because an attorney must make a reasonable inquiry into the factual contentions contained in a pleading. Answer choice D is incorrect because attorney's fees are one type of permissible sanction under Rule 11.

An account holder designated the plaintiff as the payee for an account held at a state bank. Upon the account holder's death, the plaintiff brought an action in federal court to compel the bank to pay to him the $95,000 held in the account. The bank balked at paying the plaintiff because a second claimant had informed the institution that the accountholder, prior to her death, had transferred her rights to the account to the second claimant. The plaintiff filed the action in a federal district court located in the state of which both the plaintiff and the second claimant are citizens. The bank is a citizen of another state. The bank has filed a motion to dismiss for failure to join a necessary party. Is the second claimant a necessary party? Yes, because the court has personal jurisdiction over the second claimant. Yes, because the bank could have to pay the second claimant as well as the plaintiff the amount in the account. No, because the bank can file a counterclaim and interplead the second claimant. No, because the joinder of the second claimant would defeat diversity jurisdiction.

Answer choice B is correct. A party is considered necessary if the party's absence would leave the existing parties subject to a substantial risk of multiple or inconsistent obligations. Here, the bank could be ordered to pay the $95,000 in the account to the plaintiff in this action, and to pay $95,000 to the second claimant if the second claimant were to pursue a separate action. Consequently, the second claimant is a necessary party. Answer choice A is incorrect. Whether the court has personal jurisdiction over a person who has not been joined as a party can dictate whether the court can order the joinder of the person as a party, but does not determine whether that person is a necessary party. Answer choice C is incorrect. The bank could file a counterclaim and interplead the second claimant based on federal rule interpleader because the defendant-stakeholder is not a citizen of the same state as both the plaintiff and the second claimant, and the amount-in-controversy requirement is met. The defendant, however, is not required to take this action. The existence of this option does not affect the second claimant's status as a necessary party. Answer choice D is incorrect. Whether the court lacks subject matter jurisdiction over a person who has not been joined as a party can dictate whether the court can order the joinder of the person as a party, but does not determine whether that person is a necessary party.

A plaintiff sued a defendant for personal injuries resulting from negligence in a federal district court sitting in diversity jurisdiction. The plaintiff alleged that she had been crossing at a crosswalk when the defendant ran a red light at an intersection and hit her with his truck, causing injuries. Without any showing of good cause other than the need to verify the plaintiff's injuries, the defendant filed a motion to compel the plaintiff to submit to a physical examination. The plaintiff has opposed the examination as intrusive. How should the court rule on the defendant's motion? Grant the motion, because such a physical examination is permitted if requested by the defendant, regardless of whether "good cause" is shown. Grant the motion, because the plaintiff has placed her physical condition into issue. Deny the motion, because a physical examination may only be ordered for "good cause." Deny the motion, because the court can only order a physical examination with the consent of the party to be examined.

Answer choice B is correct. Although a physical examination may be ordered only for "good cause," the plaintiff here has placed her physical condition into controversy by claiming personal injury damages, which is sufficient "good cause" for the court to order a physical examination. Answer choice A is incorrect. A court may order a physical examination of a party or a person in the legal custody or control of a party upon motion by the other party, for good cause shown. Thus, the defendant must show good cause for the examination or the court will not order the examination. Answer choice C is incorrect. It is correct that a physical examination may only be ordered for "good cause," but when the plaintiff placed her physical condition at issue by claiming personal injury damages, the requirement of "good cause" was met. Therefore, the defendant's need to verify the plaintiff's alleged injuries is considered "good cause" for ordering the examination. Answer choice D is incorrect. A party is not required to specifically consent to a physical examination in order for the court to mandate such an examination.

A jury found for the plaintiff in a defamation action in federal district court against a newspaper publisher. Following the verdict, the newspaper publisher moved for a new trial on the grounds that the verdict was against the weight of the evidence. The court granted the motion. The plaintiff wants to immediately appeal the court's order granting the new trial. Which of the following is most accurate concerning immediate appeal of the court's order? Immediate appeal is not available because the basis of the motion was that the verdict was against the weight of the evidence. Immediate appeal is precluded by the final judgment rule. Immediate appeal is available if the appeal is filed with the circuit clerk within 14 days after the order is entered. The plaintiff may appeal the order as of right.

Answer choice B is correct. An order for a new trial is not appealable because it is not a final judgment. A party who wants to raise on appeal the grant of a new trial must wait until the new trial has occurred and resulted in a final judgment. The party may then appeal from that judgment and raise as an issue the order for the new trial. Answer choices A, C, and D are incorrect because the reason that immediate appeal is not available is because the order is not a final judgment.

There was an accident involving an automobile and a truck that rendered the automobile inoperable and economically unfeasible to repair. The driver of the automobile received compensation from the insurer of the driver of the truck for the damage to the automobile and signed a release with respect to any claim for property damage against the truck driver or his insurer arising from the accident. Subsequently, the automobile driver filed a complaint in federal court against the truck driver for damages in excess of $75,000 arising from personal injuries suffered by the automobile driver. Among the statements in the complaint was the statement that the automobile was a "total loss." Twenty days after being served with the complaint and prior to serving an answer, the truck driver filed a motion to strike the "total loss" statement from the complaint. Is the court likely to grant this motion? Yes, because special damages must be specifically pled. Yes, because the statement is immaterial to the automobile driver's claim. No, because the motion was not timely filed. No, because a motion to strike may only be made with regard an insufficient defense or redundant, impertinent, or scandalous material.

Answer choice B is correct. If a pleading contains any insufficient defense, or redundant, immaterial, impertinent, or scandalous material, then the court, upon motion or upon its own initiative, may order that such defense or material be stricken. Since the statement in question concerns the value of the automobile, and the complaint only seeks to recover damages for personal injuries, the statement is immaterial. Answer choice A is incorrect because, while special damages are required to be specifically pled, property damage to an automobile stemming from a vehicular accident would be damages that normally flow from the accident, and hence not special. Moreover, even if the statement in question related to special damages, the proper motion would be a motion for a more definite statement, not a motion to strike. Answer choice C is incorrect. When a responsive pleading is permitted, the responding party must move to strike prior to responding to such a pleading. Since the motion to strike relates to the complaint, it must be made before an answer is filed, which is due within 21 days of service of the complaint. Consequently, the motion to strike in this case, which was filed 20 days after service of the complaint and before an answer was filed, was timely made. Answer choice D is incorrect because, while a motion to strike may be made for an insufficient defense or redundant, impertinent, or scandalous material, it may also be made with regard to immaterial material in a pleading, including the complaint itself.

A group of patients sought certification as a class in federal district court for a class-action lawsuit against a nursing home for damages resulting from substandard nursing care. The district court granted certification as a class, and the defendant nursing home timely and properly filed a petition for permission to appeal the certification with the appropriate court of appeals. If the court of appeals permits the nursing home's appeal, what is the effect on the district court proceedings? The proceedings in the district court are automatically stayed pending the appeal. The proceedings in the district court are stayed pending the appeal if the district court or the court of appeals so orders. The proceedings in the district court cannot be stayed pending the appeal. The proceedings in the district court can only be stayed pending the appeal by order of the district court.

Answer choice B is correct. If an appeal is permitted, it will not stay proceedings in the district court unless the district court or the court of appeals so orders. Accordingly, answer choice A is incorrect. Answer choice C is incorrect. The district court proceedings can be stayed pending the appeal if the district court or court of appeals so orders. Answer choice D is incorrect. The proceedings can be stayed pending the appeal by order of the district court or the court of appeals.

A seller of goods received two negotiable promissory notes from a buyer as consideration for a single sale of goods. One of the notes was for $50,000; the other was for $100,000. Both were due on the same date, one year from the execution of the notes. The parties anticipated that the seller would sell one of the notes at a discount shortly after their transaction in order to meet the operating needs of the seller. Instead, due to an unforeseen increase in revenue, the seller retained both notes. When the buyer defaulted on payment of the notes, the seller sued the buyer in a federal court of competent jurisdiction for failure to pay the $50,000 note and secured a final judgment against the buyer. Later, the seller sued the buyer in the same court as the first lawsuit for failure to pay the $100,000 note. The buyer contended that the seller's claim arising from the $100,000 note was barred by the seller's failure to include it in the prior action. How should the court rule on the buyer's defense to this action? For the seller, because the buyer defaulted on the $100,000 note. For the seller, because the failure to pay each note constitutes a separate, distinct cause of action. For the buyer, because the parties are the same in each lawsuit. For the buyer, because the notes were given by the buyer in payment for a single sale of goods.

Answer choice B is correct. One of the requirements imposed on the affirmative defense of claim preclusion is that the causes of action in the two lawsuits must be sufficiently identical. In applying the transactional approach to determine whether the two lawsuits in this case are sufficiently identical, although each promissory note was given by the buyer for the same sale of goods, the buyer gave two notes rather than a single one. Under these facts, the buyer was also aware that the seller intended to sell one of the notes, which would have given a third party the right to sue to enforce that note independent of the seller as holder of the other note. These facts indicate that the parties anticipated that the notes could be enforced independently, and treating the notes as a single transaction would not conform to their expectations. Consequently, the seller's failure to sue upon both of the notes in a single action does not preclude the seller from suing on each note in separate lawsuits. Answer choice A is incorrect. Although the buyer's default on the $100,000 note does give rise to the seller's cause of action, this is irrelevant with regard to the buyer's affirmative defense that the seller's failure to sue upon this note in the prior lawsuit precludes the seller from bringing a second lawsuit based on this note. Answer choice C is incorrect. Although the current lawsuit involves the same parties as the first lawsuit, according to the federal transactional approach, the causes of action were not sufficiently identical in each lawsuit. Consequently, the seller's failure to sue upon both of the notes in a single action does not preclude the seller from suing on each note in separate lawsuits. Answer choice D is incorrect. Although each promissory note arose from the same sales transaction, the ordinary commercial understanding is that the failure to pay each note constitutes a separate cause of action that can be independently pursued by the holder of the note.

A defendant in a civil action in federal district court filed a complaint against a third-party defendant for contribution. The third-party defendant moved for dismissal based on a failure to state a claim upon which relief could be granted. The court granted the motion to dismiss, without prejudice. The defendant wants to appeal immediately, contending that the trial judge erred in dismissing the complaint against the third-party defendant. Is the court's order immediately appealable? No, because of the collateral-order doctrine. No, because of the final judgment rule. Yes, because the order granted a motion to dismiss. Yes, because the order dismissed a third-party defendant.

Answer choice B is correct. Only a final judgment is subject to immediate appeal. Usually, a judge's 12(b)(6) dismissal is considered a "final judgment" subject to immediate appeal. However, when there are multiple claims and parties, any order or other decision that adjudicates fewer than all of the claims or the rights and liabilities of fewer than all of the parties will not end the action as to any of the claims or parties, and it may be revised at any time before the entry of a judgment adjudicating all of the claims and all of the parties' rights and liabilities. The district court here did not direct entry of a final judgment as to the third-party defendant. It merely dismissed the claim, without prejudice, without directing entry of a final judgment. Accordingly, answer choices C and D are incorrect. Answer choice A is incorrect. Under the collateral-order doctrine, a court of appeals has discretion to hear and rule on a district court order if it (i) conclusively determines the disputed question, (ii) resolves an important issue that is completely separate from the merits of the action, and (iii) is effectively unreviewable on appeal from a final judgment. None of these criteria are met under the facts here.

A plaintiff filed suit against a defendant in federal district court in State A under diversity jurisdiction for damages resulting from the alleged negligence of the defendant. The plaintiff is a citizen of State A and the defendant is a citizen of State B. The defendant owned a valuable antique car which he kept garaged in State C. In a State C court, the plaintiff obtained a prejudgment attachment against the defendant's car to prevent its sale before the State A lawsuit was resolved. Upon receiving notice of the attachment, the defendant moved to quash it. Which of the following statements is accurate? The prejudgment attachment is automatically unconstitutional. The attachment should be quashed if the State C court cannot constitutionally assert personal jurisdiction over the defendant. The attachment should not be quashed regardless of whether the State C court can constitutionally assert personal jurisdiction over the defendant. The federal district court in State A has in rem jurisdiction over the car.

Answer choice B is correct. Quasi-in-rem jurisdiction requires minimum contacts to exist between the defendant and the forum state before jurisdiction will apply. If the State C court cannot constitutionally assert personal jurisdiction over the defendant, as the answer choice posits, then there is no jurisdiction (either in personam or quasi-in-rem) and the attachment should be quashed. Answer choice A is incorrect because prejudgment attachment is not automatically unconstitutional. If due process is met, prejudgment attachment is constitutionally permissible. Answer choice C is incorrect because if the State C court cannot constitutionally assert personal jurisdiction over the defendant, there would be no jurisdiction (either in personam or quasi in rem) and the attachment should be quashed. Answer choice D is incorrect. In general, for in rem jurisdiction to exist, the property at issue must be present within the forum state. The car is present in State C, not State A, so the State A court would not have in rem jurisdiction. Further, jurisdiction is not based on in rem jurisdiction because the property or status of the property is not the primary object of the action, e.g., who has title. Rather, the plaintiff's claim is based on negligence and is not related to the ownership of the property that has been attached.

A trampoline company built a new model of trampoline with protective netting that was made of a cheaper material than the previous model. The company was incorporated and had its principal place of business in State A. A children's sports facility located in State B ordered these new trampolines. However, because the netting on many of the trampolines tore easily, causing a danger of children falling off the trampolines when jumping on them, the sports facility refused to pay for the trampolines. The company sued the sports facility in state court in State A, but the state court dismissed the lawsuit for lack of personal jurisdiction over the sports facility. The company then filed suit seeking payment from the sports facility for the trampolines in federal court in State A based on diversity jurisdiction. The sports facility filed a motion to dismiss based on the doctrine of res judicata. Should the court grant this motion? No, because the prior action was a state-court action. No, because the state-court dismissal was not a final judgment on the merits. Yes, because the federal-court action involves the same parties as the prior state-court action. Yes, because the federal-court action is based on the same complaint as the prior state court action.

Answer choice B is correct. Under 28 U.S.C. § 1738, federal courts must also give full faith and credit to state-court judgments. The requirement of full faith and credit extends to the res judicata effect of the original state-court judgment. Because the state court would not give res judicata effect to its prior decision dismissing a case for lack of personal jurisdiction, as it was not a final decision on the merits, the federal court is not required to do so either. Answer choice A is incorrect because, under 28 U.S.C. § 1738, federal courts must also give full faith and credit to state-court judgments. The requirement of full faith and credit extends to the res judicata effect of the original state-court judgment. In this case, because the state court would not give res judicata effect to its prior decision dismissing a case for lack of personal jurisdiction, as it was not a final decision on the merits, the federal court is not required to do so either. Answer choice C is incorrect because, even though the federal-court action involves the same parties as the prior state-court action, the state-court action was not a final judgment on the merits. Consequently, it is not entitled to res judicata effect. Answer choice D is incorrect because, even though the federal-court action is based on the same complaint as the prior state-court action, the state court action was not a final judgment on the merits.

Two women were out drinking at a bar, and after a few too many drinks, one of the women confessed to the other woman that she was the mistress of the woman's husband. The married woman was enraged and threw a glass of red wine at the mistress. In addition to ruining her expensive clothing, the wine glass also broke, and some shards embedded themselves into the mistress's right eye causing her the loss of her eyesight. The mistress sued the married woman in federal court. The court has ordered both parties to be available for a pretrial conference to consider a settlement, but the women refuse because they despise one another. Can the court direct the women or their representatives to appear at the pretrial conference? Yes, because such pretrial conferences are mandatory in cases exempt from Rule 26 disclosures. Yes, because the pretrial conference may facilitate a settlement between the women. No, because it is unlikely that the women will settle given their contentious relationship. No, because the women in good faith refused to participate in the pretrial conference.

Answer choice B is correct. Under Rule 16(a), the court may direct counsel and unrepresented parties to appear for pretrial conferences for such purposes as expediting disposition of the action, effective case management, and facilitating settlement. The court may require that a party or its representative be present or reasonably available by telephone or by more sophisticated electronic means to consider possible settlement. If counsel or a party fails to appear, fails to participate in good faith, or fails to obey a pretrial conference order, then the court may generally impose the same sanctions as those permitted for failure of a party to comply with a discovery order, including contempt of court or dismissal of an action. Here, the court can direct the women to be available for the pretrial conference because it may facilitate a settlement and expedite disposition of the action. Answer choice A is incorrect because there is no such rule. Pretrial conferences under Rule 16 may be ordered whether or not a case is subject to mandatory disclosures under Rule 26. Answer choice C is incorrect. Even if the women are unlikely to settle because they despise one another, this is not a valid reason for not being available to attempt to settle the case out of court. Answer choice D is incorrect because although the women's refusal may be in good faith, this is not a reason for not participating in the pretrial conference.

A surgeon was operating on a patient using a new device she invented. Unfortunately, the patient died. The patient's husband has filed a wrongful death lawsuit against the surgeon in federal court. The husband retained an expert witness who will testify at trial that the device used by the surgeon was defective and the cause of the patient's death. The husband disclosed the identity of this expert witness and provided the expert's report to the surgeon 30 days before the date set for trial, although the husband could have made the disclosure several months earlier. At that time, the surgeon had already hired her own expert witness to testify as to the safety of the new device. The disclosure of the husband's expert witness had no impact on the surgeon's pre-trial preparation. Can the husband's expert testify at trial? Yes, because the disclosures related to this expert were made in a timely manner. Yes, because the failure to timely make the expert disclosures was harmless. No, because the disclosures related to this expert were not made in a timely manner. No, because the timing of the disclosures was not substantially justified.

Answer choice B is correct. Under Rule 37(c)(1), if a party fails to make or supplement its automatic disclosures as required by Rules 26(a) and (e), then the party will not be permitted to use the documents or witnesses that were not disclosed unless the nondisclosure was substantially justified or was harmless. Here, the husband's expert witness disclosure was not timely. Expert witness disclosures must occur at least 90 days before the date set for trial. However, because the failure to disclose in a timely manner was harmless to the surgeon, the husband can use the testimony at trial. For this reason, answer choice C is incorrect. Answer choice A is incorrect because expert witness disclosures must occur at least 90 days before the date set for trial. Answer choice D is incorrect. Although the disclosures could have been made several months earlier and thus nondisclosure was not substantially justified, it was harmless. Therefore, the testimony could be used at trial.

A municipality was sued by a resident of the municipality in federal district court under 42 U.S.C. § 1983 for violation of his constitutional rights. The resident sought general damages of $75,000 and punitive damages of $100,000. The court required the parties to submit proposed jury instructions prior to trial. Both the resident and the municipality each submitted differing jury instructions on the issue of damages. The instructions submitted by the resident with regard to damages were ultimately given by the court to the jury, and the municipality did not object. The jury rendered a verdict, assessing both general and punitive damages against the municipality in the amounts sought by the resident. The municipality appealed the verdict. On appeal, the appellate court determined that the trial court's jury instructions were in error because punitive damages could not be awarded as a matter of law under these facts. The appellate court vacated the award of punitive damages and ordered the trial court to enter a judgment for the municipality on the issue of punitive damages. Of the following, which is the most likely reason for the appellate court's action? The appellate court determined that the trial judge abused his discretion. The appellate court applied the plain error doctrine. The trial court disregarded the municipality's proposed jury instructions. The trial court required the parties to submit proposed jury instructions before the close of evidence.

Answer choice B is correct. Unless a party timely and properly objects on the record to an erroneous instruction given or proposed to be given by the court or to the court's failure to give an instruction timely requested by a party, the party generally cannot raise the matter on appeal. However, the appellate court can address a plain error with regard to the jury instructions that affects substantial rights. Here, the trial court's mistaken instructions regarding the right of the resident to collect punitive damages from the municipality led to the award of such damages, directly affecting the rights not only of the municipality but also ultimately the taxpayers of the municipality who most likely were not directly involved in the wrongdoing but would shoulder the burden of the punitive damages award. Answer choice A is incorrect. Although the instructions provided to the jury are generally subject to the trial judge's discretion, jury instructions are generally not reviewable by an appellate court absent a proper and timely objection by a party. However, Federal Rule of Civil Procedure 51 does recognize the appellate court's right to address an erroneous jury instruction under the plain error doctrine. Answer choice C is incorrect. The court is not required to accept the proposed jury instructions of either party. Moreover, the municipality did not object to the court's instructions to the jury regarding damages. Consequently, the issue was not preserved for appeal. Therefore, the appellate court could only review the instruction on damages if there was a plain error. Answer choice D is incorrect because, under Federal Rule of Civil Procedure 51, a federal district court may require a party to submit its proposed jury instructions at any reasonable time before the close of evidence, including before trial.

A buyer purchased a residence with the aid of a loan from a relative. The relative took a mortgage on the residence as security for repayment of the loan. The buyer subsequently sold the residence to a third party. Neither the buyer nor the third party made payments that were due under the terms of the loan. The relative filed a foreclosure action in federal court against the third party based on diversity jurisdiction. The third party filed a motion to dismiss based on a lack of personal jurisdiction. The court denied this motion. Twenty days after receiving notice of the court's ruling, the third party served an answer on the relative. In the answer, the third party asserted for the first time that she recorded the deed without notice of the mortgage. The relative responded that the third party had waived this defense. Is the relative correct? Yes, because the third party failed to raise this affirmative defense in its pre-answer motion. Yes, because the third party's assertion of this defense was untimely. No, because priority under a recording act is not an enumerated affirmative defense under the Federal Rules of Civil Procedure. No, because the third party served her answer on the relative within 21 days of receiving notice of the court's action on the third party's motion to dismiss.

Answer choice B is correct. When a defendant files a pre-answer motion under Rule 12, such as a motion to dismiss for lack of personal jurisdiction, the defendant has 14 days after receiving notice of the court's decision on this motion to file its answer. Here, the defendant did not file her answer within that time period. Answer choice A is incorrect. Certain defenses (such as improper venue) must be raised along with other defenses (such as lack of personal jurisdiction) in a pre-answer motion to dismiss where such a motion is made, and are waived if subsequently raised for the first time in the answer. However, most affirmative defenses (including protection of a property interest under a recording act) may be raised for the first time by a defendant in his answer. Answer choice C is incorrect because the list of affirmative defenses enumerated in Rule 8(c) is a nonexclusive list. The applicability of the recording act to this situation is an affirmative defense because it will defeat the plaintiff's claim (i.e., the relative has a mortgage that is enforceable because the loan for which it serves as security is in default), even if the claim is true. Answer choice D is incorrect because the 21-day period in which to serve an answer applies when the defendant has not made a pre-answer motion.

A licensee of a patent brought suit against a company in federal district court for patent infringement under federal law. The company sought to join the patent holder to the action in order to avoid the possibility of multiple judgments for the same alleged patent infringement. However, the patent holder could not be joined to the action due to lack of personal jurisdiction. At the nonjury trial, the company filed a Rule 12(b) motion to dismiss for failure to join a necessary or indispensable party under Rule 19. The judge denied the company's motion to dismiss without specifying the reasons for the denial. Did the judge properly issue his ruling? No, because the judge's findings and conclusions must be stated on the record. No, because the judge must issue a written opinion of his findings and conclusions. Yes, because specific findings and conclusions are not necessary upon a court's determination of a motion to dismiss under Rule 12(b). Yes, because the judge in a nonjury trial is not required to state his findings of fact or conclusions of law unless requested by one of the parties.

Answer choice C is correct. A case will be tried by the court without a jury if no right to a jury trial exists (or if it has been waived). The court is the finder of fact; it must find the facts specially and state its conclusions of law separately. The findings and conclusions may be stated on the record after the close of the evidence or may appear in an opinion or a memorandum of decision filed by the court. The court is not required to state findings or conclusions when ruling on a motion under Rule 12 (motions against the complaint) or 56 (summary judgment) or, unless the rules provide otherwise, on any other motion. In this case, the company filed a motion to dismiss under Rule 12(b) for failure to join a necessary or indispensable party. In issuing his ruling, the judge simply denied the motion to dismiss. The judge's ruling was properly issued because he was not required to state specific findings or conclusions in his determination of the Rule 12(b) motion. Answer choice A is incorrect. In a nonjury trial, a judge is not required to state his findings and conclusions on the record; he may issue a written opinion instead. Moreover, no such statement is required upon a determination of a motion to dismiss under Rule 12(b), as in this case. Answer choice B is incorrect because a judge may state his findings and conclusions on the record, rather than in a written opinion. However, the judge was not required to state his findings and conclusions here. Answer choice D is incorrect. In a nonjury trial, a judge must generally state his findings of fact and conclusions of law on the record or in a written opinion, even if not requested to do so by one of the parties, unless the determination involves a motion under Rule 12.

Without justification, a plaintiff failed to appear for trial in federal district court. The trial had been postponed several times, and the court had warned the parties that failure to proceed would lead to dismissal of the case. Upon the defendant's motion, the court properly dismissed the case without stating whether the dismissal was with or without prejudice. At the time of dismissal, 130 days had passed since the case was first set for trial. Is the plaintiff entitled to pursue a new action based on the same claim in federal court? Yes, because the first dismissal of an action in federal court is without prejudice. Yes, because the presumption is that dismissal of the case by court order is without prejudice. No, because the plaintiff failed to prosecute his case. No, because more than 90 days had passed since the case was first set for trial.

Answer choice C is correct. If a plaintiff fails to prosecute his case and, in response to a defendant's motion, the court dismisses the action, the dismissal is with prejudice and operates as an adjudication on the merits. Answer choice A is incorrect. While a voluntary dismissal of an action by a plaintiff is usually without prejudice if undertaken before the opposing party serves either an answer or a motion for summary judgment or if all other parties agree, this rule does not apply when the defendant is the party seeking a dismissal. Answer choice B is incorrect because, as noted with regard to answer choice C, the presumption is that dismissal of a case by the court upon a defendant's motion is with prejudice. Answer choice D is incorrect. Although there is a federal rule that requires service of process within 90 days of the filing of a complaint, there is no similar rule regarding the prosecution of a case.

A patient from State A traveled to State B for a minor medical procedure with a well-known doctor from State C. After the procedure was complete, the doctor failed to properly stitch the area where he had made an incision. As a result, the patient contracted a major infection. The patient filed a diversity action for medical malpractice against the doctor in federal district court in State C, alleging $300,000 in damages. During discovery, the doctor filed a motion to transfer venue to State B. After reviewing the doctor's motion, the court determined that State B was a more appropriate venue and transferred the case. What choice-of-law rules should the court in State B apply? State A State B State C Either State B or State C

Answer choice C is correct. In a diversity case, if a court transfers the case, then the district court to which the case is transferred must apply the law that would have been applied in the district court that transferred the case. This rule only applies when venue is proper in the original court. In this case, venue was proper in State C, the state where the doctor resides. However, the district court in State C transferred the case to State B, another proper venue. Because the action was transferred from State C, the district court in State B must apply the choice-of-law rules, which are considered substantive, of State C. Answer choice A is incorrect because the court in State B must apply the law of the transferor court. Answer choice B is incorrect. Although a federal district court sitting in diversity jurisdiction would generally apply the substantive law of the state in which it is located, the fact that the case was transferred from State C means that State C substantive law applies. Answer choice D is incorrect. The court in State B does not have the option of deciding which choice-of-law rules to apply; it must apply the law of State C, the transferor court.

A defendant filed a complaint against a third-party defendant for contribution permitted under federal law for any environmental damages for which the defendant was found liable. The third-party defendant and the defendant were domiciled in the same state. The third-party defendant filed a motion to dismiss for failure to state a claim upon which relief could be granted. The third-party defendant submitted affidavits in support of its position, but the court refused to consider them. After taking all well-pleaded facts in the complaint as true and resolving all doubts and inferences in the defendant's favor, the court denied the motion to dismiss. Has the court acted properly in making its ruling? No, because the court lacked subject-matter jurisdiction over the complaint against the third-party defendant. No, because the court failed to consider the third-party defendant's affidavits. Yes, because the court took all well-pleaded facts in the complaint as true and resolved all doubts and inferences in the defendant's favor. Yes, because a third-party defendant cannot file a motion to dismiss for failure to state a claim upon which relief can be granted.

Answer choice C is correct. In ruling on a motion to dismiss for failure to state a claim upon which relief can be granted, the court must take all well-pleaded facts in the complaint as true and resolve all doubts and inferences in the complainant's favor. Here, the defendant is the complainant. Answer choice A is incorrect because the third-party defendant's motion was not a motion to dismiss for lack of subject-matter jurisdiction, but rather a motion to dismiss for failure to state a claim upon which relief can be granted. Moreover, although a court may on its own motion raise the issue of lack of subject-matter jurisdiction, the court has subject-matter jurisdiction over the defendant's contribution complaint, despite the lack of diversity of citizenship, because it is based on federal law. Answer choice B is incorrect. In ruling on a motion to dismiss for failure to state a claim upon which relief can be granted, the court may consider only the allegations in the complaint, any exhibits attached to the complaint, and any matters subject to judicial notice. (Note that the court could consider outside material like an affidavit for a motion for summary judgment.) Answer choice D is incorrect because a third-party defendant, as with any party against whom a claim is asserted, may file a motion to dismiss for failure to state a claim upon which relief can be granted.

A plaintiff, a citizen of State X, sued a defendant, a citizen of State Y, for negligence in federal district court in State X under diversity jurisdiction, in connection with an automobile accident that occurred in State Y. The defendant has had no contacts with State X. The plaintiff personally served the defendant with a summons and complaint at his home in State Y. The defendant's first response to the complaint was an answer that specifically denied the plaintiff's claims but omitted the defense of lack of personal jurisdiction. Fifteen days after serving the answer on the plaintiff, the defendant amended the answer to include the defense of lack of personal jurisdiction without asking leave of the court. Which of the following statements is most accurate regarding the defendant's actions? The defense of lack of personal jurisdiction was permanently waived when the defendant's answer failed to include it. The defense of lack of personal jurisdiction has been waived unless the court subsequently grants the defendant leave to amend his answer. The defense of lack of personal jurisdiction was not waived and may be asserted by the defendant. The defense of lack of personal jurisdiction can never be waived and may always be asserted by a defendant.

Answer choice C is correct. Pursuant to Rule 15(a)(1)(a), a party may amend its pleading once as a matter of course within 21 days after serving it. Under Rule 12(h)(1)(B), a party waives the defense of lack of personal jurisdiction by failing to include it in a pre-answer motion, a responsive pleading, or in an amendment allowed by Rule 15(a)(1), as a matter of course. Here, the defendant included the defense of lack of personal jurisdiction in an amendment made within 21 days after serving its answer. Accordingly, the defense was not waived and may be asserted by the defendant in the case. Answer choice A is incorrect. The failure to include the defense of lack of personal jurisdiction in the defendant's answer would have waived the defense, but the defendant subsequently amended his answer within 21 days of serving it to include the defense. Answer choice B is incorrect. Because the defendant amended the answer within 21 days of serving it, leave of court is not required to amend, pursuant to Rule 15(a)(1)(a). Answer choice D is incorrect. The defense of lack of personal jurisdiction (unlike lack of subject matter jurisdiction) can be waived, as discussed above.

A famous politician brought a federal diversity action against a newspaper for libel. The newspaper had printed an article describing the famous politician as "a crook who can be easily bribed." At trial, the famous politician's lawyer delivered her opening statement, followed by the opening statement of the newspaper's lawyer. The judge then adjourned the trial for the day. The newspaper's lawyer immediately filed a motion for judgment as a matter of law. The motion specified the judgment sought and detailed all of the facts behind the newspaper's statement in the article, as well as the law supporting judgment in its favor. Was the newspaper's motion proper? Yes, because it was supported by the specific facts and law entitling the newspaper to the judgment. Yes, because the motion was made prior to the presentation of all evidence at trial. No, because the famous politician had not been fully heard on any issue. No, because the newspaper had not presented its case to the jury.

Answer choice C is correct. Under Rule 50(a), once a party has been fully heard on an issue at a jury trial, the court may grant a motion for judgment as a matter of law resolving the issue against a party, if the court finds that there is insufficient evidence for a jury to reasonably find for that party. In this case, the parties had only delivered their opening arguments. The famous politician had not yet presented evidence on any issue in the case. Therefore, the newspaper's motion was premature and thus improper. Answer choice A is incorrect. A motion for judgment as a matter of law must specify the judgment sought and the law and facts that entitle the movant to the judgment. In this case, the newspaper met this requirement. However, the motion was not filed at an appropriate time in the trial. Answer choice B is incorrect. A motion for judgment as a matter of law may be made at any time before the case is submitted to the jury, provided a party has been fully heard on an issue. Thus, the motion may be made after the presentation of all evidence but before the case goes to the jury. Answer choice D is incorrect. The newspaper does not need to first present its case to the jury before being able to file a motion for judgment as a matter of law. It may properly file a motion for judgment as a matter of law after the famous politician has been fully heard on an issue (typically, at the close of the famous politician's case).

A plaintiff brought an action in federal district court to compel the owner of real property to sell the property to the plaintiff pursuant to their agreement. The plaintiff, knowing that there was "bad blood" between the owner and his adult son, approached the son about serving process on his father. The son readily agreed. Although the plaintiff had intended for the owner's son to serve the owner personally, he instead served the owner by certified mail. The procedural rules of the forum state do not permit service by certified mail, but the procedural rules of the state in which the owner now resides permit such service. The owner filed a motion to quash service. How should the court rule on the owner's motion? Grant the motion, because the owner was served with process by his son. Grant the motion, because the procedural rules of the forum state do not permit service by certified mail. Deny the motion, because the procedural rules of the state in which service was made permit service by certified mail. Deny the motion, because the plaintiff had intended for the owner to be personally served.

Answer choice C is correct. Under the federal rules, service may be made by following state law for serving a summons in an action brought in courts of general jurisdiction in the state where the district court is located (the forum state) or in the state where service is made. Because service by certified mail is permitted under the rules of the state where the owner was served with process, the service of process in this case was proper. Therefore, the court should deny the owner's motion to dismiss. Answer choice A is incorrect because service of process may be made by any nonparty who is at least 18 years old, even a member of the defendant's family. Answer choice B is incorrect because, as noted with regard to answer choice C, the federal rules, in addition to permitting service of process in accord with the forum state, also permit service of process in accord with the state where service is made. Answer choice D is incorrect. Because the means by which the owner was served were proper, it is irrelevant that the plaintiff intended for the process server to use another means.

A plaintiff brought a defamation action in federal district court based on diversity jurisdiction. Following the presentation of all evidence in the case, the court submitted the case to the jury. The jury, after being properly instructed, was supplied with a verdict form that contained both specific interrogatories as well as a general verdict. In answering the interrogatories, the jury found that the plaintiff was a public figure and that the defendant did not act with actual malice. The jury's general verdict awarded the plaintiff $200,000 in damages. Based on the jury's answers to the specific interrogatories, the court, concluding that, by law, the plaintiff could not be awarded damages, approved for entry by the clerk a judgment in favor of the defendant. Is the court's action proper? No, because it is at odds with the jury's general verdict. No, because a jury that has been supplied with special interrogatories cannot also render a general verdict. Yes, because it is in accord with the jury's answers to the special interrogatories. Yes, because the court must approve a judgment in accord with the jury's answers to the interrogatories.

Answer choice C is correct. When a jury is instructed to deliver both a general verdict and to answer special interrogatories, and the answers are consistent with each other but not with the general verdict, the court may (i) approve a judgment that is consistent with the answers, notwithstanding the general verdict, (ii) direct the jury to reconsider its answers and verdict, or (iii) order a new trial. In this instance, the jury, through its answers to the special interrogatories, found that the plaintiff was a public figure and that the defendant did not act with actual malice. By law, if a plaintiff is a public figure and the defendant does not act with actual malice, the defendant is not liable for defamation. Therefore, the court could approve a judgment in favor of the defendant that was in accord with the jury's answers to the special interrogatories. Answer choice A is incorrect. Generally, a court, unless ruling on a party's renewed motion for judgment as a matter of law, may not order the entry of a judgment that is contrary to a jury's general verdict. However, when the jury's answers to special interrogatories contradict the jury's general verdict, the court may order a judgment that is in accord with those answers. Answer choice B is incorrect because Federal Rule of Civil Procedure 49 specifically permits the use of special interrogatories along with a general verdict. Answer choice D is incorrect. In this circumstance, the court may (i) approve a judgment that is consistent with the answers, notwithstanding the general verdict, (ii) direct the jury to reconsider its answers and verdict, or (iii) order a new trial. The court is not required to approve a judgment consistent with the answers; rather, the court has discretion.

A plaintiff filed an action against a defendant in federal district court. The complaint alleged that the defendant had infringed upon a trademark held by the plaintiff under federal law and sought $55,000 in damages. In addition, the plaintiff claimed damages of $30,000 allegedly attributable to the defendant's negligence in causing an auto accident involving the two parties. The plaintiff and the defendant are citizens of different states. The defendant moved to dismiss the negligence claim for lack of subject matter jurisdiction. The court denied the motion. Is the court's ruling correct? Yes, because the court can exercise supplemental jurisdiction over the negligence claim. Yes, because the court has diversity jurisdiction over the negligence claim. No, because neither diversity nor supplemental jurisdiction exists with respect to the negligence claim. No, because a state law based claim may not be joined with a claim over which the court has federal question jurisdiction.

Answer choice C is correct. While the court has subject matter jurisdiction over the trademark infringement claim based on federal question jurisdiction, the court does not have subject matter jurisdiction over the negligence claim. The negligence claim is based on state, not federal law; therefore, the court does not have federal question jurisdiction over the claim. In addition, although the diversity-of-citizenship requirement is satisfied because the parties are citizens of different states, the amount-in-controversy requirement is not satisfied because the plaintiff is seeking damages of only $30,000. The negligence claim does not arise out of the same "common nucleus of operative fact" as the trademark infringement claim, so supplemental jurisdiction over the negligence claim would also not be appropriate. Consequently, the court should grant the defendant's motion to dismiss the negligence claim. Answer choice A is incorrect. The negligence claim does not arise out of the same "common nucleus of operative fact" as the trademark claim, so supplemental jurisdiction over the negligence claim would not be appropriate. Answer choice B is incorrect. Although the diversity-of-citizenship requirement is satisfied because the parties are citizens of different states, the amount-in-controversy requirement is not satisfied. A plaintiff's claims cannot be aggregated to meet the amount-in-controversy requirement when the original claim is based on federal question jurisdiction. Answer choice D is incorrect.

A company properly filed a diversity action in federal district court against an accountant for malpractice in preparing the corporation's financial statements. The accountant impleaded his malpractice insurer, asserting a claim against the insurer in the event that the accountant was liable to the company. The company then timely filed a third-party claim against the insurer for $90,000 in damages to the company vehicles caused by a falling tree. The insurer and the accountant are citizens of the same state, while the company is a citizen of a different state. The insurer has moved to dismiss this claim due to improper joinder. Should the court grant the insurer's motion? No, because the court has subject matter jurisdiction over the company's claim against the insurer. No, because a plaintiff may file a third-party claim against a third-party defendant. Yes, because the court lacks jurisdiction over the claim asserted by the accountant against the insurer. Yes, because the company's claim against the insurer does not arise out of the same transaction or occurrence as the original complaint.

Answer choice D is correct. A plaintiff may file a third-party claim against a third-party defendant, but the claim must fall under the court's jurisdiction and relate to the same transaction or occurrence as the original complaint. Because the company's claim against the insurer does not relate to the company's malpractice claim against the accountant, the company is not permitted to assert this claim as part of the current lawsuit. Answer choice A is incorrect. The court does have subject matter jurisdiction over the company's claim against the insurer. However, despite the existence of subject matter jurisdiction, the rules do not permit the company to join this unrelated claim. Answer choice B is incorrect. Although a plaintiff may file a third-party claim against a third-party defendant, this claim must relate to the same transaction or occurrence as the original complaint. Answer choice C is incorrect because the court has subject matter jurisdiction over the claim asserted by the accountant against the insurer. Diversity jurisdiction does not exist, as both the accountant and the insurer are citizens of the same state. However, the court can exercise supplemental jurisdiction over this claim because it arises out of the same transaction or occurrence as the original complaint.

A class action lawsuit filed in federal court was certified under Rule 23(b)(2). The complaint alleged that the defendant had engaged in gender discrimination against the class in its hiring practices, in violation of federal law. The complaint sought injunctive relief. To what type of notice is a member of the class who is not a named representative entitled? The same notice as a named class representative. The best notice that is practicable under the circumstances. Individual notice, by personal delivery or registered mail. Appropriate notice, at the discretion of the court.

Answer choice D is correct. For a class action that is certified under Rule 23(b)(1) or (2), notice to the class members, other than the named representatives, is at the discretion of the court. The court may order that appropriate notice be given, which often takes the form of publication notice. Answer choice A is incorrect because a class member who is not a named representative is not treated as a party to the action for purposes of notification, but instead is subject to special notification rules. Answer choice B is incorrect. It states the standard set forth in the rule for member of a class that is certified under Rule 23(b)(3), and is inapplicable here. Answer choice C is incorrect. Although a class member who is not named as a representative is generally entitled to individual notice if the class was certified under Rule 23(b)(3), this type of notice is not required when the class is certified under Rule 23(b)(1) or (2). Moreover, even in the case of members of a class certified under Rule 23(b)(3), notice is not typically provided by means of personal delivery or registered mail.

A patient filed a medical malpractice action against a physician in federal court based on diversity jurisdiction. At the close of the patient's case to the jury, the physician's attorney moved for a judgment as a matter of law that the patient had failed to establish the physician's negligence. The court denied this motion. The patient's attorney did not move for a judgment as a matter of law at that time. At the close of the physician's case to the jury, the patient's attorney moved for judgment as a matter of law that the physician's affirmative defense based on the expiration of the statute of limitations had not been established. The court determined that no reasonable jury could find that the statute of limitations had expired. Can the court grant the motion of the patient's attorney? No, because the patient's attorney did not move for a judgment as a matter of law at the close of the plaintiff's case. No, because a judgment as a matter of law cannot be rendered with regard to a defense. Yes, because a motion for a judgment as a matter of law can be made at any time before the jury renders a verdict. Yes, because the motion made by the patient's attorney was timely and proper.

Answer choice D is correct. In a jury trial, a party may move for a judgment as a matter of law with regard to a defense as well as a claim. A party may make such a motion once the opposing party has been fully heard on the issue. Since the physician's attorney has presented his case, the motion by the patient's attorney with regard to the physician's statute of limitations defense was both proper and timely. Answer choice A is incorrect. A party cannot move for a judgment as a matter of law with respect to a claim or defense until the other party has been fully heard on that matter. Since the physician's attorney had not had the opportunity to present the physician's case at the close of the patient's case, a motion made at that time with regard to an affirmative defense would have been improper. Answer choice B is incorrect because Rule 50 permits the court to entertain a motion for a judgment as a matter of law with regard to defenses as well as claims. Answer choice C is incorrect because it misstates the deadline for making a motion for a judgment as a matter of law. This motion must be made prior to the submission of the case to the jury.

A plaintiff, who resides in a western state with a single federal judicial district, brought suit against the United States under the Federal Tort Claims Act for an assault and battery committed by a federal employee in the same state. The suit was filed in the U.S. District Court for the District of Columbia. The United States attorney did not file a pre-answer motion to dismiss. In her answer, the United States attorney asserted that venue is improper. Should the court recognize this defense? No, because improper venue must be raised by a pre-answer motion. No, because the United States is a defendant. Yes, because venue is proper only in the judicial district in which the plaintiff resides. Yes, because venue is improper.

Answer choice D is correct. In an action filed under the Federal Tort Claims Act, venue is proper either in the judicial district where the plaintiff resides or in the judicial district where the act or omission occurred. Here, the judicial district for the western state in which the plaintiff resides is also the district in which the act or omission that gave rise to the claim arose. Consequently, the U.S. District Court for the District of Columbia is not the proper court in which to bring this suit. (Note: The Federal Tort Claims Act waives sovereign immunity and permits a tort action against the United States under limited circumstances.) Answer choice A is incorrect because, although the defense of improper venue may be raised in a pre-answer motion under Rule 12(b)(3), it may be raised in the answer if such a motion is not filed. Answer choice B is incorrect because the Federal Tort Claims Act specifies venue in the judicial district in which the plaintiff resides or where the act or omission occurred; there is no alternate jurisdiction in the DC District Court. Answer choice C is incorrect because, while the court should recognize the improper venue defense, in a claim brought under the Federal Tort Claims Act, venue is proper either in the judicial district in which the plaintiff resides or in the judicial district in which the act or omission occurred.

A homeowners association sought a temporary restraining order (TRO) against a member of the association to prevent the building a structure on her property in violation of rules adopted by the association. In seeking the order, the attorney for the association falsely certify in writing that he had been unsuccessful in his efforts to give the member notice of the TRO request. The court ex parte granted the TRO for 14 days. When the member received notice of the TRO the following day, she filed an appeal, challenging the propriety of the court's grant of the TRO. Should the appeals court dismiss this appeal? No, because the TRO was issued ex parte. No, because the attorney for the association had not made an effort to give the member notice of the TRO request. Yes, because only the denial of a TRO is appealable. Yes, because the TRO is an interlocutory order.

Answer choice D is correct. In general, a TRO is not an appealable interlocutory order unless it is extended beyond the period permit under Rule 65. Under Rule 65, a court is generally permitted to grant a TRO for a period of no more than 14 days in the absence of the existence of good cause or the consent of the adversary. Here, since the TRO is effective for 14 days, the granting of the TRO in question cannot be appealed. In contrast, a TRO that is extended beyond this time limit will be treated as a preliminary injunction and therefore be appealable. Answer choice A is incorrect. A court may grant a TRO on an ex parte basis when the moving party can establish, under written oath, that immediate and irreparable injury will result prior to hearing the adverse party's opposition and the movant's attorney certifies in writing any efforts made to give notice and the reason why notice should not be required. Although a criterion for the issuance of a TRO was falsely established, the granting of TRO is not immediately appealable because such an order is an interlocutory order. Answer choice B is incorrect because, although a criterion for the issuance of a TRO was falsely established, the granting of TRO is not immediately appealable because such an order is an interlocutory order. Answer choice C is incorrect because generally neither the denial nor the granting of a TRO is an appealable interlocutory order.

Answer choice D is correct. Under Rule 56, a motion for summary judgment must be granted if the pleadings, the discovery and disclosure materials on file, and any affidavits show that there is no genuine dispute as to any material fact and that the movant is entitled to judgment as a matter of law. A genuine issue of material fact exists when a reasonable jury could return a verdict in favor of the nonmoving party. In ruling on a motion for summary judgment, the court must construe all evidence in the light most favorable to the nonmoving party and resolve all doubts in favor of the nonmoving party. Here, the court did not resolve the issue regarding how the patron, the nonmoving party, had ingested sugar at the coffee store in a light most favorable to the patron. Therefore, summary judgment was not proper. Answer choice A is incorrect. Although it states the correct standard, there is a genuine dispute as to how the sugar was ingested by the patron at the coffee shop when this issue is viewed in the light most favorable to the patron. Answer choice B is incorrect. If the court had properly resolved the issue of how the patron ingested the sugar in the light most favorable to the patron, it would have found that a reasonable jury arguably could return a favorable verdict for the patron because the issue was not clear. Answer choice C is incorrect. In deciding a motion for summary judgment, the court must consider the materials cited by the parties—affidavits, documents, electronically stored information, pleadings, deposition transcripts, interrogatory answers, admissions, and stipulations filed by the party, even if not presented in a form that is admissible at trial, as long as the facts contained in the submissions are admissible at trial. Yes, because there was no genuine dispute as to any material fact in the case. Yes, because a reasonable jury could not return a favorable verdict for the patron. No, because the evidence was not presented in a form admissible at trial. No, because the court did not resolve all doubts in favor of the nonmoving party.

Answer choice D is correct. Under Rule 56, a motion for summary judgment must be granted if the pleadings, the discovery and disclosure materials on file, and any affidavits show that there is no genuine dispute as to any material fact and that the movant is entitled to judgment as a matter of law. A genuine issue of material fact exists when a reasonable jury could return a verdict in favor of the nonmoving party. In ruling on a motion for summary judgment, the court must construe all evidence in the light most favorable to the nonmoving party and resolve all doubts in favor of the nonmoving party. Here, the court did not resolve the issue regarding how the patron, the nonmoving party, had ingested sugar at the coffee store in a light most favorable to the patron. Therefore, summary judgment was not proper. Answer choice A is incorrect. Although it states the correct standard, there is a genuine dispute as to how the sugar was ingested by the patron at the coffee shop when this issue is viewed in the light most favorable to the patron. Answer choice B is incorrect. If the court had properly resolved the issue of how the patron ingested the sugar in the light most favorable to the patron, it would have found that a reasonable jury arguably could return a favorable verdict for the patron because the issue was not clear. Answer choice C is incorrect. In deciding a motion for summary judgment, the court must consider the materials cited by the parties—affidavits, documents, electronically stored information, pleadings, deposition transcripts, interrogatory answers, admissions, and stipulations filed by the party, even if not presented in a form that is admissible at trial, as long as the facts contained in the submissions are admissible at trial.

A university student sued the university for damages arising from an accident that occurred during a chemistry lab experiment. The student testified that the accident had permanently blinded her. She also presented evidence from her doctor that she would never see again. The jury rendered a verdict for the student and awarded her $10 million in damages. Three months after the judgment was entered, the university received evidence that last week the student was driving a car in which the doctor was a passenger. Which of the following is the most likely means by which the university can secure relief from the judgment? File a new trial motion because the verdict was based on false evidence. File a motion to correct the judgment because the damages awarded were clearly excessive. File a motion for relief from the judgment based on the newly discovered evidence that the student was not permanently blind. File a motion for relief from the judgment based on intrinsic fraud by the student.

Answer choice D is correct. Under Rule 60(b), a court may relieve a party from a final judgment or order for a motion filed, within a reasonable time, and no later than one year following the entry of the judgment or order for mistake, inadvertence, surprise, excusable neglect, newly discovered evidence that could not have been earlier discovered with reasonable diligence, fraud, misrepresentation, or misconduct by an opposing party. Under this rule, both intrinsic fraud, which is fraud relates to a matter tried by the trier of fact, and extrinsic fraud, which is fraud that prevents a person from discovering or enforcing legal rights, can serve as grounds for relief. Here, the new evidence suggests that the student and her doctor colluded to present false evidence regarding at the very least the permanency of her blindness. Since this evidence relates to a matter tried by the trier of fact, it is intrinsic fraud, and as such, can serve as grounds for relief from the judgment. Note that it is likely that the university, by promptly filing a motion for relief, will have acted within a reasonable time and certainly will not exceed the outer one-year limit for filing this motion. Answer choice A is incorrect because, although a ground for filing a new trial motion is that the verdict is based on false evidence, a new trial motion must be filed no later than 28 days after the entry of judgment. Here, this motion would exceed that time limit. Answer choice B is incorrect because, while a motion to correct a judgment under Rule 60(a) is not subject to a time limitation, such a motion allows a court to correct a clerical or other mistake resulting from oversight or omission whenever one is found in a judgment, order, or other part of the record. Here, there is no such clerical or similar mistake in the judgment. Answer choice C is incorrect because, although as noted regarding answer choice A, a motion for relief from a judgment under Rule 60(b) is permitted for newly discovered evidence, the newly discovered evidence must be evidence that could not have been earlier discovered with reasonable diligence. This restriction implies that the evidence itself must have existed at the time of the trial. Since the evidence in question relates to an event that took place almost three months after judgment was entered, it arguably does not constitute newly discovered evidence for purposes of this rule.

A mayor sued a blogger for defamation in federal district court under diversity jurisdiction. The mayor alleged in her complaint that the blogger had published defamatory statements about her that suggested she was having an adulterous relationship. The mayor's entire case rested on her own testimony establishing the prima facie elements of her claim and a properly authenticated and admitted copy of the allegedly defamatory publication. At the end of the mayor's presentation of evidence to the jury, the blogger filed a motion for judgment as a matter of law. Finding that the mayor's meager evidence was insufficient for a jury reasonably to find that the publication was false, as was required by state law, the judge granted the blogger's motion and directed a judgment in favor of the blogger. The mayor immediately appealed the judgment, contending that the trial judge applied the wrong legal standard in granting the motion. On these facts, should the judgment be set aside on appeal? No, because the district court's ruling was not clearly erroneous. No, because the mayor failed to meet her burden of establishing a prima facie case as a matter of law. Yes, because a motion for judgment as a matter of law cannot be granted until both parties have presented their cases. Yes, because the district court improperly evaluated the weight of the evidence.

When ruling on a Rule 50 motion for judgment as a matter of law, the court must view the evidence in the light most favorable to the opposing party and draw all reasonable inferences from the evidence in favor of the opposing party. It may not consider the credibility of witnesses or evaluate the weight of the evidence, and it must disregard all evidence favorable to the moving party that the jury is not required to believe. Therefore, on these facts, the district court improperly granted the blogger's motion, and the ruling should be set aside. Answer choice A is incorrect because it states the incorrect standard to be applied when reviewing a judgment as a matter of law. Appellate review of legal rulings is de novo. The appeals court will use the trial court's record, but it reviews the evidence and law without deference to the trial court's rulings. Answer choice B is incorrect because the mayor met this burden by testifying that the publication was false. The credibility of this testimony must be assessed by the jury. Answer choice C is incorrect because a motion for judgment as a matter of law may be made at any time before the case is submitted to the jury.

The plaintiff, a resident of State A, was involved in an automobile accident with the defendant, who is a resident of State B. The plaintiff filed a negligence claim against the defendant in federal district court seeking to obtain $80,000 in damages. At the time of the accident, a passenger was riding in the plaintiff's car. The passenger, a resident of State B, also suffered injuries and seeks to join in the plaintiff's claim alleging $50,000 in damages. Does the federal district court have jurisdiction over the passenger's claim? Yes, because the passenger's claim and the plaintiff's claim arise out of a common nucleus of operative fact. Yes, because an additional claim need not meet the amount-in-controversy requirement for federal diversity jurisdiction. No, because allowing joinder of the passenger's claim would destroy complete diversity. No, because the passenger's claim does not meet the amount-in-controversy requirement for federal diversity jurisdiction.

nswer choice C is correct. Although a federal district court with jurisdiction over a claim may exercise supplemental jurisdiction over additional claims that arise out of a common nucleus of operative fact, the addition of a party cannot defeat complete diversity. Here, the defendant is a resident of State B. Because the passenger is also a resident of State B, adding the passenger's claim would defeat complete diversity. Answer choice A is incorrect because, although it states the correct standard for the federal district court's assertion of supplemental jurisdiction, it ignores the requirement that the parties must maintain complete diversity. Answer choice B is incorrect. Although it is true that an additional claim need not meet the amount-in-controversy requirement for federal diversity jurisdiction, the parties must maintain complete diversity. Answer choice D is incorrect because it is an incorrect statement of law. Additional claims are not required to satisfy the amount-in-controversy requirement for purposes of supplemental jurisdiction.


Ensembles d'études connexes

Orbit Anatomy, Orbits and Facial bone Anatomy

View Set

Diagnosis+Psychopathology- Exam Qs

View Set